Negatives Potential unendlicher quadratischer Brunnen

Ein 1D-Well mit endlichem Quadrat wird im Allgemeinen entweder durch definiert

(1) v ( X ) = { 0 A X A v 0 ansonsten
oder
(2) v ( X ) = { v 0 A X A 0 ansonsten

Wo A > 0 Und v 0 > 0 . Es geht nur darum, den Nullpunkt Ihres Potenzials zu definieren.

Unendlich quadratisch kann auch als Grenze angesehen werden v 0 + von ( 1 ) , dh

(3) v ( X ) = { 0 A X A + ansonsten

In diesem Fall ist die Wellenfunktion außerhalb des Brunnens Null und Sie lösen die Schrödinger-Gleichung innerhalb unter Anwendung von Randbedingungen.

Was ist mit dem Limit von ( 2 ) ?

(4) v ( X ) = { A X A 0 ansonsten

Da wir es jetzt mit Unendlichkeiten zu tun haben, bin ich mir nicht sicher, ob ich sagen soll ( 3 ) Und ( 4 ) unterscheiden sich nur durch die Wahl des Potentials. Außerdem, was kann ich über die Wellenfunktion innen und außen in dem Fall sagen ( 4 ) ?

Antworten (1)

Eine gute Möglichkeit, Probleme mit "Unendlichkeiten" anzugehen, besteht darin, sie für endliche Werte zu untersuchen / zu lösen und dann eine geeignete Grenze zu nehmen. Hier könnte man also schreiben

v ( X ) = lim v 0 v v 0 ( X ) = lim v 0 { 0 X [ A , A ] v 0 ansonsten
und lösen Sie die Schrödinger-Gleichung und nehmen Sie dann den Grenzwert.

Dies impliziert, dass Sie die Eigenschaften der Lösung der Schrödinger-Gleichung für eine Nicht-Unendlichkeit übertragen können v 0 . Insbesondere die Unabhängigkeit von der Wahl der Nullstelle des Potentials (z. B. die Invarianz unter beliebigen Verschiebungen v ( X ) v ( X ) + C Ö N S T ), was impliziert, dass (3) und (4) äquivalent sind. Wenn Sie die Wellenfunktionen sorgfältig studieren, werden Sie sehen, dass sie eine konstante Phase erhalten, während Sie die Potenzialverschiebung vornehmen. Aber diese Phase ist ohnehin nicht beobachtbar (man erinnere sich, dass Zustände tatsächlich Elemente eines projektiven Hilbert-Raums sind). Alle beobachtbaren Eigenschaften bleiben unter dieser potentiellen Verschiebung invariant, insbesondere das (Nicht-)Verschwinden der Intensität | ψ | 2 innerhalb/außerhalb des Brunnens.

Wenn ( 3 ) Und ( 4 ) gleichwertig sind, dann zu studieren ( 4 ) ohne die Grenze zu überschreiten, würde man sagen, dass die Wellenfunktion außerhalb des Brunnens genauso Null sein muss wie innerhalb ( 3 ) . Ist das korrekt?